LSAT and Law School Admissions Forum

Get expert LSAT preparation and law school admissions advice from PowerScore Test Preparation.

User avatar
 Dave Killoran
PowerScore Staff
  • PowerScore Staff
  • Posts: 5848
  • Joined: Mar 25, 2011
|
#84952
Complete Question Explanation
(The complete setup for this game can be found here: lsat/viewtopic.php?f=357&t=12842)

The correct answer choice is (D)

The conditions in the question stem produce the following initial diagram:

G3-Q18-d1.png

From the second and third rules, the fifth bead cannot be R or O:

G3-Q18-d2.png

At this point, the inference stream appears to stop. So, where should you look to help solve the problem? The key here (as in #16) is to apply the space-eating first rule. If the fifth and sixth beads are P and Y, then the seventh bead will be R. From the third rule the eighth bead cannot be O, which will ultimately cause a violation of the fourth rule because there will be no room for O among the first eight beads. Hence, P and Y cannot be fifth and sixth, and answer choice (D) is correct.

Get the most out of your LSAT Prep Plus subscription.

Analyze and track your performance with our Testing and Analytics Package.